A Test for Absolute Convergence of a Series

In summary, a series is a mathematical concept that involves adding together a sequence of numbers or terms. Absolute convergence is a property of a series where the sum of its terms converges to a finite value, regardless of the order in which the terms are added. To test for absolute convergence, the Cauchy's root test is commonly used. Absolute convergence is important because it guarantees that the series will converge to a finite value, regardless of the order of the terms. A series can exhibit both absolute and conditional convergence, where it converges absolutely for some order of the terms and conditionally for others.
  • #1
Euge
Gold Member
MHB
POTW Director
2,072
240
Let ##\{a_n\}_{n = 1}^\infty## be a sequence of real numbers such that for some real number ##p > 1##, ##\frac{a_n}{a_{n+1}} = 1 + \frac{p}{n} + b_n## where ##\sum b_n## converges absolutely. Show that ##\sum a_n## also converges absolutely.
 
  • Like
Likes topsquark
Physics news on Phys.org
  • #2
If a sequence divergences to ##+ \infty## then so does every subsequence. For any ##r > 0##, we must have that ##\frac{r}{n} \leq |b_n|## for only a finite number of terms, otherwise ##\infty = \sum^\infty \frac{r}{n} \leq \sum^\infty |b_n|## where the sum is taken over an arbitrary subsequence. Therefore, there exists a ##N## such that ##\frac{r}{n} > |b_n|## for all ##n > N##. Therefore, there exists a ##N## such that ##\frac{r}{n} + b_n > 0## for all ##n > N##.

Define a ##q## such that ##1 < q < p##. There exists an ##N## such that ##\frac{p-q}{n} + b_n > 0## for all ##n > N##. From ##\frac{a_n}{a_{n+1}} = 1 + \frac{p}{n} + b_n## we have

\begin{align*}
\dfrac{|a_n|}{|a_{n+1}|} & = |1 + \frac{q}{n} + \frac{p-q}{n} + b_n|
\nonumber \\
& > 1 + \frac{q}{n}
\end{align*}

for ##n > N##.

Rearranged:

\begin{align*}
n \left( \dfrac{|a_n|}{|a_{n+1}|} - 1 \right) > q \qquad (*)
\end{align*}

The Raabe-Duhamel's test: Let ##\{ c_n \}## be a sequence of positive numbers. Define

\begin{align*}
\rho_n := n \left( \dfrac{c_n}{c_{n+1}} - 1 \right)
\end{align*}

if

\begin{align*}
L = \lim_{n \rightarrow \infty} \rho_n
\end{align*}

exists and ##L > 1## the series converges.

From ##(*)## we have

\begin{align*}
L = \lim_{n \rightarrow \infty} \rho_n = \lim_{n \rightarrow \infty} n \left( \dfrac{|a_n|}{|a_{n+1}|} - 1 \right) > q > 1 .
\end{align*}

Hence, ##\sum |a_n|## converges.
 
Last edited:
  • Like
Likes topsquark

Similar threads

  • Math POTW for University Students
Replies
1
Views
2K
  • Math POTW for University Students
Replies
1
Views
190
  • Math POTW for University Students
Replies
1
Views
839
  • Calculus and Beyond Homework Help
Replies
1
Views
504
  • Calculus and Beyond Homework Help
Replies
2
Views
437
Replies
5
Views
193
  • Calculus and Beyond Homework Help
Replies
3
Views
233
  • Math POTW for University Students
Replies
1
Views
545
  • Calculus and Beyond Homework Help
Replies
8
Views
1K
  • Calculus and Beyond Homework Help
Replies
2
Views
897
Back
Top